Difference between revisions of "2002 AMC 12B Problems/Problem 9"

(New page: The answer is 1/4. The arithmetic sequence doesn't require much thought as it is 1,2,3,4. The geometric sequence is 1,2,4.)
 
Line 1: Line 1:
The answer is 1/4.
+
==Problem==
 +
If <math>a,b,c,d</math> are positive real numbers such that <math>a,b,c,d</math> form an increasing arithmetic sequence and <math>a,b,d</math> form a geometric sequence, then <math>\frac ad</math> is
  
The arithmetic sequence doesn't require much thought as it is 1,2,3,4.
+
<math>\mathrm{(A)}\ \frac 1{12}
 +
\qquad\mathrm{(B)}\ \frac 16
 +
\qquad\mathrm{(C)}\ \frac 14
 +
\qquad\mathrm{(D)}\ \frac 13
 +
\qquad\mathrm{(E)}\ \frac 12</math>
  
The geometric sequence is 1,2,4.
+
==Solution==
 +
We can let a=1, b=2, c=3, and d=4. <math>\frac{a}{d}=\frac{1}{4} -> \boxed{\mathrm{(C)}}</math>
 +
 
 +
==See also==

Revision as of 09:48, 5 February 2008

Problem

If $a,b,c,d$ are positive real numbers such that $a,b,c,d$ form an increasing arithmetic sequence and $a,b,d$ form a geometric sequence, then $\frac ad$ is

$\mathrm{(A)}\ \frac 1{12} \qquad\mathrm{(B)}\ \frac 16 \qquad\mathrm{(C)}\ \frac 14 \qquad\mathrm{(D)}\ \frac 13 \qquad\mathrm{(E)}\ \frac 12$

Solution

We can let a=1, b=2, c=3, and d=4. $\frac{a}{d}=\frac{1}{4} -> \boxed{\mathrm{(C)}}$

See also